Use the diagram shown to find 4 ÷ 1/3

Answers

Answer 1

Answer:

12

Step-by-step explanation:

Since there is no diagram. I will just tell you the answer. first convert into multiplcation keep everything but the 1/3. Change 1/3 into 3/1 (3). Multiply 4*3=12.

Sorry if I don't have a graph.

Hope this helps!


Related Questions

PLZ I NEED HELPPPPPPPP

Answers

answers are 5a) C 5b) C

Eighty members of a bike club were asked whether they like touring bikes and whether they like mountain bikes. A total of 70 like touring bikes, 47 like mountain bikes, and 5 do not like either.

A 4-column table with 3 rows. The first column has no label with entries likes mountain bikes, does not like mountain bikes, total. The second column is labeled liking touring bikes with entries a, c, 70. The third column is labeled does not like touring bikes with entries b, 5, e. The fourth column is labeled total with entries 47, d, 80.

What are the correct values of a, b, c, d, and e?

a = 42, b = 28, c = 33, d = 5, e = 10
a = 42, b = 5, c = 28, d = 33, e = 10
a = 5, b = 42, c = 28, d = 10, e = 33
a = 5, b = 10, c = 28, d = 33, e = 42

Answers

Answer: Choice B

a = 42, b = 5, c = 28, d = 33, e = 10

====================================================

Explanation:

Refer to the diagram below. We have a table of values in which some values (if not most) are unknown. We have variables as placeholders until we can figure out which numbers replace them.

Along the bottom row, we see that 70+e = 80, which solves to e = 10. I subtracted 70 from both sides.

That must mean the answer is between choice A or choice B.

--------------------------

Since e = 10, and b+5 = e (third column), we can replace the 'e' with 10 to get the equation b+5 = 10. That solves to b = 5. This rules out choice A.

The final answer is choice B

---------------------------

If you want to keep going to find a, c and d, then let's solve a+b = 47 which is the same as a+5 = 47 after replacing letter b with 5.

a+5 = 47 solves to a = 42 after subtracting both sides by 5.

Then along the first column, we see that a+c = 70 which is the same as 42+c = 70. Isolating c gets us c = 28

Lastly, c+5 = d is shown along the second row. Plug in c = 28 to find that d = c+5 = 28+5 = 33

So overall we have:

a = 42, b = 5, c = 28, d = 33, e = 10

Answer:

b is correct

Step-by-step explanation:

Consider a maximization linear programming problem with extreme points xi, x2, Xz. and x4. and extreme directions d1,. d2, and dz. and with an objective function gradient e such that cx1 =4, cx2 = 6, cx3= 6, cx4=3, cd1= 0, cd2=0, and cd3=2. Characterize the set of alternative optimal solutions to this problem.

Answers

Answer:

Set of alternative optimal solution : 0 ≤ z ≤ 1.5

Hence There will be an infinite set of Alternative optimal solution

Step-by-step explanation:

considering Cx1 = 4

∴ C = 4 / x1

Cx2 = 6

∴ 4x2 - 6x1  = 0

2x2 - 3x1 = 0 ------ ( 1 )

considering Cx3 = 6

C = 6/x3

Cx4 = 3

∴ (6/x3) x4 - 3 = 0

= 2x4 - x3 = 0 ---- ( 2 )

attached below is the remaining part of the solution

set of alternative optimal solution : 0 ≤ z ≤ 1.5

There will be an infinite set of Alternative optimal solution

plsssss help it’s timed!!!!!!

Answers

Answer:

the answer to this question is 36.86989°

find the surface area of the composite figure​

Answers

Answer:

[tex]=280[/tex] [tex]in^2[/tex]

Step-by-step explanation:

----------------------------------------

Let's find the surface area of the pink rectangular prism first.

[tex]2*10=20+20=40[/tex]

[tex]4*10=40+40=80[/tex]

[tex]4*2=8+8=16[/tex]

[tex]40+80+16=136[/tex]

The surface area for the pink rectangular prism is [tex]136[/tex] [tex]in^2[/tex].

-------------------->>>>>

Now, let's find the surface area of the green rectangular prism.

[tex]4*7=28+28=56[/tex]

[tex]4*7=28+28=56[/tex]

[tex]4*4=16+16=32[/tex]

[tex]56+56+32=144[/tex]

The surface area for the green rectangular prism is 144 [tex]in^2[/tex].

-------------------->>>>>

Now let's add the surface area of both rectangular prisms to find the surface area of the composite figure.

[tex]136+144=[/tex]

[tex]=280[/tex] [tex]in^2[/tex]

----------------------------------------

Hope this is helpful.

9514 1404 393

Answer:

  224 in²

Step-by-step explanation:

There are a couple of ways to go at this. Here, we choose to figure the areas of each of the prisms individually, then subtract the "hidden" area where they are joined together.

The area of a prism is ...

  A = 2(LW +H(L+W))

Pink area:

  A = 2(10·4 +2(10+4)) = 2(40 +28) = 136 . . . square inches

Green area:

  A = 2(7·4 +4(7+4)) = 2(28 +44) = 144 . . . square inches

One 4 in × 7 in face of the green prism meets with a similar area of the pink prism, so the area hidden at that interface is 2(4·7) = 56 square inches. Then the total surface area of the composite figure is ...

  SA = 136 in² +144 in² -56 in² = 224 in²

Rationalize the denominator of the fraction and enter the new denominator below.​

Answers

Answer:

7/19

Step-by-step explanation:

7/19=square root of 11=22-3 19

Rewrite the given equation in logarithmic form. Then, select all of the equations with an equivalent solution.
8e^x - 5 = 0

Answers

Answer:

ans: ln (5/8) , ln5 - ln8

Step-by-step explanation:

8e^x -5 = 0

e^x = 5/8

x = ln (5/8)

x = ln5 - ln8

What is a number divided by 3
gives a remainder of 1, divided by 4
gives a remainder of 2, divided by
5 gives a remainder of 3?

Answers

Answer:

58

Step-by-step explanation:

58/3 gives a remainder of 1

58/4 gives a remainder of 2

58/5 gives a remainder of 3

If the coordinates of a point p(m-3 , -6) = p(-7 , -6), then find the value of m .

Answers

Answer:

[tex]m =-4[/tex]

Step-by-step explanation:

Given

[tex]p(m-3 , -6) = p(-7 , -6)[/tex]

Required

Find m

[tex]p(m-3 , -6) = p(-7 , -6)[/tex]

By comparison:

[tex]m-3 = -7[/tex]

Add 3 to both sides

[tex]m = -7+3[/tex]

[tex]m =-4[/tex]

Help me with this question

Answers

Answer:

D. The graph of g(x) is the graph of f(x) compressed vertically and then reflected over the x axis.

Step-by-step explanation:

A function shows the relationship between two or more variables.

If a function y = f(x) is reflected over the x axis, the x coordinate remain unchanged, but the y coordinate is negated. Hence the function y = f(x) becomes y = -f(x).

A function y = f(x) is compressed or stretched vertically by a factor k to give y = kf(x). If 0 < k < 1, the function is vertically compressed whereas if k > 1, the function is vertically stretched.

Given the function f(x) = x², the function is vertically compressed by a factor of 2/3 to form a function f(x)' = (2/3)x². The function is then reflected over the x axis to produce a function g(x) = (-2/3)x²

Analyze the graph below and complete the instructions as follows.

Answers

Answer:

Option A:

x^2 + (y - 2)^2 = 9

Step-by-step explanation:

We know that the equation for a circle centered in the point (a, b) and of radius R is given by:

(x - a)^2 + (y - b)^2 = R^2

So the first thing we need to find is the center of the circle.

We can see that the center is at:

x = 0

y = 2

Then the center is at the point (0, 2)

Now we want our circle to pass through point 2, located at a distance of 2 units from the radius of the first circle.

So the distance between the center and point 2 is 2 units plus the radius of the smaller circle:

And the radius of the smaller circle is one unit.

Then, the radius of a circle centered at (0, 2) that passes through point 2 is:

R = 1 + 2 = 3

Then we have a circle centered at (0, 2) and of radius R = 3

Replacing these in the equation for a circle we get:

(x - 0)^2 + (y - 2)^2 = 3^2

x^2 + (y - 2)^2 = 9

The correct option is A

The hypotenuse of a right triangle is twice the length of one of its legs. The length of the other leg is three feet. Find the lengths of the three sides of the triangle. Enter an exact answer. Do not type your answers as fractions or decimals

Answers

Answer:

One leg: 3

So, the hypo: 2*3 = 6

And the third is according to the formula for right angle triangles: a^2 + b^ = c^2

So: √9+36= c

6.7 to get an exact number round it: 7

Answer:

sides are √3 ft,2√3 ft,3 ft

Step-by-step explanation:

let one leg=x

length of hypotenuse=2x

third side=3 ft

(2x)²=x²+3²

4x²-x²=9

3x²=9

x²=9/3=3

x=√3 ft

hypotenuse=2√3 ft

please help me please help me please help me please help me please help me please help me please​

Answers

Answer:

q5 is 4

q6 is 72

Step-by-step explanation:

yan na po ..sana maktulong sau

1st question- 4 digits
2nd question- 72

graph the
function f(x)=10(2)x

Answers

Answer:

G.o.o.g.l.e

Step-by-step explanation:

If you search up 'f(x)=10(2)x' on g.o.o.g.l.e it will draw the graph for you.

If this helps you, please give brainliest!

What is tan 0 when csc 0= 2/3

Answers

Answer:

[tex]\tan{\theta} = \frac{\sqrt{11}}{11}[/tex]

Step-by-step explanation:

Cosecant:

The cosecant is one divided by the sine. Thus:

[tex]\csc{\theta} = \frac{1}{\sin{\theta}}[/tex]

Tangent is sine divided by cosine, so we first find the sine, then the cosine, to find the tangent.

Sine and cosine:

[tex]\sin{\theta} = \frac{1}{\csc{\theta}} = \frac{1}{2\sqrt{3}} \times \frac{\sqrt{3}}{\sqrt{3}} = \frac{\sqrt{3}}{6}[/tex]

[tex]\sin^{2}{\theta} + \cos^{2}{\theta} = 1[/tex]

[tex]\cos^{2}{\theta} = 1 - \sin^{2}{\theta}[/tex]

[tex]\cos^{2}{\theta} = 1 - (\frac{\sqrt{3}}{6})^2[/tex]

[tex]\cos^{2}{\theta} = 1 - \frac{3}{36}[/tex]

[tex]\cos^{2}{\theta} = \frac{33}{36}[/tex]

First quadrant, so the cosine is positive. Then

[tex]\cos^{2}{\theta} = \sqrt{\frac{33}{36}} = \frac{\sqrt{33}}{6}[/tex]

Tangent:

Sine divided by cosine. So

[tex]\tan{\theta} = \frac{\sin{\theta}}{\cos{\theta}} = \frac{\frac{\sqrt{3}}{6}}{\frac{\sqrt{33}}{6}} = \frac{\sqrt{3}}{\sqrt{33}} = \frac{\sqrt{3}}{\sqrt{3}\sqrt{11}} = \frac{1}{\sqrt{11}} \times \frac{\sqrt{11}}{\sqrt{11}} = \frac{\sqrt{11}}{11}[/tex]

The answer is:

[tex]\tan{\theta} = \frac{\sqrt{11}}{11}[/tex]

Please help me I am confused and i will give you anything you want just help me. SOS

Answers

Answer:

hope it helps you..........

Evaluate the given equation for the indicated function values. pls help

Answers

Answer:

The answer in each numeral is:

f(4) = 28f(10) = -19f(-5) = -33f(9) = -9

Step-by-step explanation:

To obtain the result in each case, you must replace the variable (n) by the value that appears in the second case, I'll explain it with the first exercise:

1. f(n) = 5n + 8   f(4) = ?

As you can see, in the second doesn't appear f(n), but f(4), that means you must replace the "n" in the equation by 4, if we do this, we obtain:

1. f(4) = 5*(4) + 8f(4) = 20 + 8f(4) = 28

The first answer is 28, now we'll continue with the next exercises:

2. f(n) = -2n + 1f(10) = -2*(10) + 1f(10) = -20 + 1f(10) = -19

3. f(n) = 6n - 3f(-5) = 6*(-5) - 3f(-5) = -30 - 3f(-5) = -33

4. f(n) = -nf(9) = -9

In this form, you can prove the answers are: 28, -19, -33, and -9 respectively.

What is 15 5/7 - 6 4/5

Answers

Answer:

8.9

Step-by-step explanation:

15.71428571-6.8=8.914285714

We round of to one significant figure because its addition n the lowest is 6.8

Answer:

[tex]8\frac{32}{35}[/tex]

Step-by-step explanation:

Identify two segments that are marked congruent to each other on the diagram
below. (Diagram is not to scale.)
K
H
#
is congruent to

Answers

Answer:

segments LJ and LI are congruent

Step-by-step explanation:

look for the little lines (tick marks)

similar marks mean congruent to each other

The two congruent segments in the figure are LJ and LI.

What is congruency?

The Side-Angle-Side Congruence Theorem (SAS) defines two triangles to be congruent to each other if the included angle and two sides of one is congruent to the included angle and corresponding two sides of the other triangle.

An included angle is found between two sides that are under consideration. Thus, two triangles having two pairs of corresponding sides and one pair of corresponding angles that are congruent to each other is not enough justification for proving that the two triangles are congruent based on the SAS Congruence Theorem.

In the figure, the two segments marked are LI and LJ these two segments are congruent to each other. Congruency means the shape and the size of the segments will be equal to each other.

To know more about congruency follow

https://brainly.com/question/2938476

#SPJ2

A) Which inequality is shown on this graph


B) which graph shows the inequality



Image attached

Answers

A for both, the first options

Please mark brainliest

expression 6 times 50

Answers

Answer:

6x50=300

Step-by-step explanation:

because math

Answer:

6*50

HOPE THIS HELPS

- Todo ❤️

Step-by-step explanation:

6*5=30*10=300

f(x) = 4 - 2x – 2x3
g(x) = x² + 7x-9
Find f(x) + g(x).

Answers

Answer:

-2x^3+x^2+5x-5

Step-by-step explanation:

f(x) = 4 - 2x – 2x^3

g(x) = x² + 7x-9

f(x) + g(x)=4 - 2x – 2x^3+ x² + 7x-9

Combine like  terms

f(x) + g(x) = -2x^3+x^2+5x-5

What side is the shortest in the picture?

A. GF
B. DG
C. EF
D. GE
F. DE

Answers

Answer:

A. GF

Step-by-step explanation:

The shortest side in a triangle is opposite the smallest angle

<d = 180 -52 -61 =67

The smallest angle is 52 so the smallest side is DG

<f = 180 - 48-85 =50

The smallest angle is 48 so the smallest side is FG

The smallest angle is 48 so the smallest side overall is FG (GF)

What is the length of leg s of the triangle below?
459
872
90+
45
A. 8
B. v
C. 8-12
D. 4.2
E. 1
F. 2

Answers

Answer:

option A

Step-by-step explanation:

take 45 degree as reference angle

using sin rule

sin 45 = opposite / hypotenuse

[tex]\frac{1}{\sqrt{2} }[/tex] = s/[tex]8\sqrt{2}[/tex]

[tex]\frac{1}{\sqrt{2} } *8\sqrt{2}[/tex] = s

root 2 and root 2 gets cancel

8 = s

Consider the given functions. Select the expression that will produce h(x). A. f(x) + f(x) B. f(x) − g(x) C. f(x) + g(x) D. g(x) − f(x)

Answers

Answer:

here is your answer

Step-by-step explanation:

here is your answer

Can someone help me please???!

Answers

Answer:

1) Yes, it is a right angle triangle

2)Yes, it is a right angle triangle

3) No, they are not similar.

Step-by-step explanation:

Dimension of triangle A = 48, 55 & 73

Dimension of triangle B = 36, 77 & 85

For any of the triangles to be a right angled one, then;

c = √(a² + b²)

Where a,b & c are side dimensions of a triangle.

Thus;

Triangle A: c = √(48² + 55²)

c = √5329

c = 73

This tallies with what we are given and so it is a right angled triangle.

Triangle B: c = √(36² + 77²)

c = √(7225)

c = 85

Similar to the third side dimension of 85, thus it is true.

For Triangle A & B to be similar, the ratio of the 3 corresponding sides must be in a whole number ratio.

Thus, we have;

48/36 = 1.5

55/77 = 5/7

73/85 = 73/85

Since the ratios are not similar, then we can say that the triangles are not similar.

HW HELP ASAP PLZZZZZ

Answers

Hello,

3x³a + 3x²a²

common factor : 3x²a

3x³a + 3x²a²

= 3x²a * x + 3x²a * a

= 3x²a(x + a)

:-)

Step-by-step explanation:

hope this will help you more

Which function is graphed?

Answers

Answer:B

Step-by-step explanation:

This is the only possible answer trust me

Find the investment value when compounded anually.
P = $120,000, r= 5.3%, t = 8 yr​

Answers

Given:

[tex]P=\$120,000[/tex]

[tex]r=5.3\%[/tex]

[tex]t=8\text{ years}[/tex]

To find:

The value of the investment when the interest is compounded annually.

Solution:

The formula for amount is:

[tex]A=P\left(1+\dfrac{r}{n}\right)^{nt}[/tex]

Where, P is the principal, r is the rate of interest in decimal, n is the number of time interest compounded in an years, and t is the number of years.

The interest is compounded annually. So, [tex]n=1[/tex].

Substituting [tex]P=120000, r=0.053, n=1, t=8[/tex] in the above formula, we get

[tex]A=120000\left(1+\dfrac{0.053}{1}\right)^{1(8)}[/tex]

[tex]A=120000\left(1.053\right)^{8}[/tex]

[tex]A=181387.85936[/tex]

[tex]A\approx 181387.86[/tex]

Therefore, the value of the investment after 8 years is $181,387.86.

Which of the data sets below has a mean of 48? Select all that apply.

A) 51, 53, 43
B) 24, 91, 18, 65, 52
C) 65, 18, 72, 33, 52
D) 72, 18, 56, 46

Answers

C) 65, 18, 72, 33 and D) 72, 18, 56, 46
Other Questions
What do you think influences the development of characters in a family? Check all that apply. Find the missing value The diagram below represents the net of a triangular prism.36 cm12 cm6 cm8 cm10 cm6 cm(not drawn to scale]What is the surface area of the triangular prism?A288 square centimetersB312 square centimeters336 square centimetersD368 square centimeters Which is most likely the healthiest serving of meat? gii thch th no l gic i,gic dt, gic ngoi xm What are the mensure of 1 angle and 2 angle? Show ur work or explain A supreme figure is required in every organization who can reward or punish the members for their deed. In the absence of such a figure, do you think that issues like laziness and irresponsibility can be resolved on its own? Support your answer with proper reasons in context with the lesson How the Camel got his Hump. Substance A decomposes at a rate proportional to the amount of A present. a) Write an equation that gives the amount A left of an initial amount A0 after time t. b) It is found that 8 lb of A will reduce to 4 lb in 4.6 hr After how long will there be only 1 lb left? a) Choose the equation that gives A in terms of A0, t, and k, where k > 0.b) There will be 1 lb left after 14 hr (Do not round until the final answer. Then round to the nearest whole number as needed.) List 3 physical side effects of anger/stress and explain how they can impact your driving Which sentence from the excerpt shows the narrator is using logos the mean age of 8 people is 16 years. when mrs. hernandez's age is included, the mean age increases to 20. how old is mrs. hernandez? QUESTION 3 3.1.1 If y= 2x2 - 1. What is the value of y when x = -2? How was the Enlightenment idea of popular sovereignty reflected in the U.S.government after the American Revolution?A. The Declaration of Independence identified the British king as anunfit sovereign.B. The Bill of Rights declared that the federal government could notcontrol the states.C. The U.S. Constitution created a legislative house that would beelected directly by the people.D. The Articles of Confederation granted Congress control over themilitary and the postal service.LI Who holds concurrent powers?a. the federal governmentb. the state governmentsc. both the federal government and state governmentsd individual citizens Which of the following is the equation of a line in slope-intercept form for aline with slope = and yintercept at (0, -1)?O A. y - x-1B. y = 4x-1O c. y= |x-1O D. y=x+1 PLZ HELP BC IF I DONT FINISH THIS IN TIME I WONT PASS MY CLASS Trust incurred $10,000 of portfolio income. Its corporate trustee paid fiduciary fees of $1,000 therefrom, and also paid $1,000 in premiums for a life insurance policy on Marcia, the grantor of the trust. How much gross income does Marcia include with respect to these trust activities?A) $800. B) $1,000. C) $8,000. D) $9,000. E) $10,000. A mixture of 0.5940 M CO and 0.3950 M Cl2 is enclosed in a vessel and heated to 1000 K . CO(g)+Cl2(g)COCl2(g)c=255.0 at 1000 K Calculate the equilibrium concentration of each gas at 1000 K Which of the following acids has a basicity of 2?1 pointethanoic acidhydrochloric acidnitric acidsulphuric acid PLEASE HELP THIS IS AN EMERGENCY!!!!A template of a Venn diagram representing common and differentiating characteristics of covalent and ionic bonds is shown.Which of the following characteristics can be written only in space A? (5 points)A.)Formed between atoms of similar electronegativitiesB.)Formed between two metal atoms mostly found on the left of the periodic tableC.)Formed between two non-metal atoms mostly found on the right of the periodic tableD.)Formed between an atom that tends to lose electrons and an atom that tends to gain electrons